Guest | Sign in | Register

Sign in to LucentBlackBoard

E-mail Id
Password
Loading...

New Member to LucentBlackBoard?

Forgot Your Password?

Enter your registered e-mail address, your password will be sent to e-mail address.
Enter E-mail Id
Loading...

Time left :





Note:

1. Total number of questions : 15.

2. Time alloted : 20 minutes.

3. Each question carry 1 mark, no negative marks.

4. Click the 'Submit Test' button given in the bottom of this page to Submit your answers.

5. Test will be submitted automatically if the time expired.

6. Don't refresh the page.

Result and Statistics

Time Left :

Result and Statistics ( Test No : online_test_number )

A. Number Of Question

15

B. Number Of Attempted

-

C. Number Of Correct Answer

-

D. Number Of Wrong Answer ( D = B-C )

-

E. Total Score ( E = C )

-

F. Accuracy Rate ( F = C / B * 100 )

-

G. Total Percentage ( G = C / A * 100 )

-



1.

Average of 11 numbers is 50. If the average of the first six numbers is 49 and that of last six is 52, what is the sixth number?

A.    87 Wrong Right

B.    56 Wrong Right

C.    87 Wrong Right

D.    78 Wrong Right

Answer : 56

Explanation : Total of the 11 numbers= 11*50 =550
Total of the first six numbers = 6*49 = 294
The total of the list six numbers=6 * 52 = 312

Therefore the Sixth number = 294 + 312 – 550 = 56

Learn more problems on : Average

Discuss on this question

2.

The banker’s gain on a bill due 2 years hence at 10% per annum is Rs.10. What is the true discount?

A.    25 Wrong Right

B.    50 Wrong Right

C.    100 Wrong Right

D.    75 Wrong Right

Answer : 50

Explanation :

Learn more problems on : Banker's Discount

Discuss on this question

3.

A flagstaff 17.5 m high casts a shadow of length 40.25 m. The height of the building, which casts a shadow of length 28.75 m under similar conditions will be

A.    10 m Wrong Right

B.    12.5 m Wrong Right

C.    17.5 m Wrong Right

D.    21.25 m Wrong Right

Answer : 17.5 m

Explanation : Let the height of the building be x metres.
Less lengthy shadow, less is the height (Direct proportion)

Hence, 40.25 : 28.75 :: 17.5 : x
           40.25 * x = 28.75 * 17.5

           x = (28.75 * 17.5)/ 40.25
           x = 12.5

Learn more problems on : Chain Rule

Discuss on this question

4.

The angle between the minute hand and the hour hand of a clock when the time is 4.20 is

A.    Zero Degree Wrong Right

B.    10 degrees Wrong Right

C.    5 degrees Wrong Right

D.    20 degrees Wrong Right

Answer : 10 degrees

Explanation : Angle traced by hour hand in 17/2 hours = (369/12 * 17/2) = 130 degrees
Angle traced by minute hand in 20 min = (360/60 * 20) = 120 degrees
Hence, Required angle = (130 – 120) degrees = 10 degrees

Learn more problems on : Clock

Discuss on this question

5.

The bacteria in a culture grow by 5% in the first hour, get destroyed by 5% in the second hour and again grow b 5% in the third hour. If the count of the bacteria at the end of the third hour is 8.379 * 108 , find the original count of bacteria in the sample.

A.    8*106 Wrong Right

B.    8*104 Wrong Right

C.    8*107 Wrong Right

D.    8*108 Wrong Right

Answer : 8*108

Explanation : Let the original count be x. After the 1st hour , the count is x(1+5/100) = 1.05x. After the 2nd hour , the count is 1.05x(1 – 5/100) = 1.05(0.95)x. After the 3rd hour, the count is [1.05(0.95)x]( 1 + 5/100) = (1.05)(0.95) (1.05)x. Therefore, x= 8*108

Learn more problems on : Compound Interest

Discuss on this question

6.

 The number 51,80,00,000 when expressed in scientific notation equals?

A.    51.8 * 106                     Wrong Right

B.    51.8 * 107 Wrong Right

C.    5.18 * 108                      Wrong Right

D.    5.18 * 109 Wrong Right

Answer : 5.18 * 108                     

Explanation : 51,80,00,000 = 5.18 * 100000000 = 5.18 * 108

Learn more problems on : Decimal Fraction

Discuss on this question

7.

A, B and C jointly thought of engaging themselves in a business venture. It was agreed that A would invest Rs. 6500 for 6 months, B, Rs. 8400 for 5 months and C, Rs. 10,000 for 3 months. A wants to be the working member for which, he was to receive 5% of the profits. The profit earned was Rs. 7400. Calculate the share of B in the profit. 

A.    Rs. 1900 Wrong Right

B.    Rs. 2660 Wrong Right

C.    Rs. 2800 Wrong Right

D.    Rs. 2840 Wrong Right

Answer : Rs. 2660

Explanation : For managing, A received = 5% of Rs. 7400 = Rs. 370.
Balance = Rs. (7400 - 370) = Rs. 7030.
Ratio of their investments = (6500 x 6) : (8400 x 5) : (10000 x 3)
= 39000 : 42000 : 30000
= 13 : 14 : 10
B's share = Rs. 7030 x (14/37)   = Rs. 2660.

Learn more problems on : Partnership

Discuss on this question

8.

When the price of a commodity is increased by x%, then the percent by which the consumption should be reduced so as not to increase the expenditure is given by:
{x/ (100 + x)} * 100 %
For x = 15
(15/ 115) * 100 = 300/23
x% of y + y% of x = ?

A.    2% of xy Wrong Right

B.    50% of xy Wrong Right

C.    2xy Wrong Right

D.    100% of xy Wrong Right

Answer : 2% of xy

Explanation : x% of y = x/100 * y = xy/100
Similarly, y% of x = xy/100
Addition of the two gives 2 * xy/100 = 2/100 xy = 2% of xy

Learn more problems on : Percentage

Discuss on this question

9.

Three taps A, B and C can fill a tank in 12, 15 and 20 hours respectively. If A is open all the time and B and C are open for one hour each alternately, the tank will be full in: 

A.    6 hours Wrong Right

B.    6 2/3 hours Wrong Right

C.    7 hours Wrong Right

D.    7 1/2 hours Wrong Right

Answer : 7 hours

Explanation :

Learn more problems on : Pipes and Cistern

Discuss on this question

10.

Two friends A and B apply for a job in the same company. The chances of A getting selected is 2/5 and that of B is 4/7. What is the probability that both of them get selected?

A.    8/35  Wrong Right

B.    34/35 Wrong Right

C.    27/35 Wrong Right

D.    None of these Wrong Right

Answer : 8/35 

Explanation : P(A) = 2/5
P(B) = 4/7
E = {A and B both get selected}
P(E) = P(A)*P(B)
= 2/5 * 4/7
= 8/35

Learn more problems on : Probability

Discuss on this question

11.

The present ages of three persons in proportions 4 : 7 : 9. Eight years ago, the sum of their ages was 56. Find their present ages (in years). 

A.    8, 20, 28 Wrong Right

B.    16, 28, 36 Wrong Right

C.    20, 35, 45 Wrong Right

D.    None of these Wrong Right

Answer : 16, 28, 36

Explanation : Let their present ages be 4x, 7x and 9x years respectively.
Then, (4x - 8) + (7x - 8) + (9x - 8) = 56
20x = 80
x = 4.
Their present ages are 4x = 16 years, 7x = 28 years and 9x = 36 years respectively.

Learn more problems on : Problems on Ages

Discuss on this question

12.

3 bells beep at an interval of 12, 20, and 35 minutes. If they beep together at 10 a.m., then they will again beep together at:

A.    12 p.m. Wrong Right

B.    1 p.m. Wrong Right

C.    4 p.m. Wrong Right

5 p.m"/> D.    5 p.m Wrong Right

Answer : 5 p.m

Explanation : The L.C.M. of 12, 20 and 35 is 420. Hence, all 3 bells beep together after 420 minutes = 7 hours
Hence, the 3 bells will beep together 7 hours after 10 a.m. i.e. at 5 p.m.

Learn more problems on : Problems on H.C.F and L.C.M

Discuss on this question

13.

A train of 24 carriages, each of 60 meters length with an engine of 60 meters length is running at a speed of 60 km/hr. find out the time within which the train will cross the bridge measure 1 ½ km in length

A.    4 minutes Wrong Right

B.    7 minutes Wrong Right

C.    3 minutes Wrong Right

D.    10 minutes Wrong Right

Answer : 3 minutes

Explanation : length of the train = 24(60) + 60 = 1500 m.
=> Total distance to be covered = 1.5 + 1.5 = 3km
=> Time taken = 3/60 = (3/60)60 = 3 minutes

Learn more problems on : Problems on Trains

Discuss on this question

14.

A sold a table to B at a profit of 15%. Later B sold it back to A at a profit of 20% thereby gaining Rs. 69. How much did A pay for the table originally?

A.    Rs. 350 Wrong Right

B.    Rs. 450 Wrong Right

C.    Rs. 405 Wrong Right

D.    Rs. 300 Wrong Right

Answer : Rs. 300

Explanation : Let Cost Price of A = 100; Selling Price of A = 100(115 / 100) = Rs. 115
Cost Price of B = Rs. 115; Selling Price of B = 115(120/100) = Rs. 138
= Cost of Price A
Gain of B = 138 – 115 = Rs. 23
If Cost Price of A = 100, gain of B = 23
Gain of B = 69
Cost Price of A = (100 × 69)/23 = Rs. 300

Learn more problems on : Profit and Loss

Discuss on this question

15.

A can do a work in 32 days and B in 40 days. They worked together for 10 days and then B leaves the work. In how many days will A alone finish the remaining work?

A.    12 days Wrong Right

B.    10/3 days  Wrong Right

C.    17.5 days Wrong Right

D.    14 days Wrong Right

Answer : 14 days

Explanation : A’s one day work = 1/32 and B’s one day work = 1/40
(A+B)’s one day work = 1/32 + 1/40 = 5+4/160 = 9/160
(A+B)’s 10 days work = 9 × 10 / 160 = 9/16
Remaining work = 1 – 9/16 = 7/16
A will finish the work in 7/16 / 1/32 = 14 days.

Learn more problems on : Time and Work

Discuss on this question

Partner Sites
LucentBlackBoard.com                  SoftLucent.com                  LucentJobs.com
All rights reserved © 2012-2015 SoftLucent.